LSAT and Law School Admissions Forum

Get expert LSAT preparation and law school admissions advice from PowerScore Test Preparation.

 Administrator
PowerScore Staff
  • PowerScore Staff
  • Posts: 8917
  • Joined: Feb 02, 2011
|
#65990
Please post your questions below!
 HK_Official
  • Posts: 3
  • Joined: Jun 15, 2020
|
#88906
“The author of the passage would be most likely to agree with which one of the following statements regarding pollution caused by fish farming?”

So it's pretty clear we need to go to ¶2 for support. I get to C and E, and I'm unsure how to decide between the two . How can we justify the conditional in C?

For reference:
C - Such pollution is less likely to occur if fish farming enterprises do not pursue intensive production methods.
E - Such pollution is, however harmful, unlikely to prevent fish farming from eventually relieving the pressure on wild fishery stocks.
User avatar
 Bob O'Halloran
PowerScore Staff
  • PowerScore Staff
  • Posts: 61
  • Joined: Jul 06, 2021
|
#88944
Hi HK_Official,
Thank you for your question.
Answer choice (C) is supported by the first sentence of paragraph 2 -
"In the first place, the more intensive forms of fish farming, oriented toward high-volume production,
threaten the sustainability of ocean fisheries....."
Also, answer choice (E) uses convoluted language but is actually saying that pollution will not prevent fish farming from eventually relieving the pressure on wild fishery stocks.
The author believes that pollution will play a part in preventing fish farming from relieving the pressure.
Let us know if you have other questions.
Bob
User avatar
 aadhiswaran
  • Posts: 4
  • Joined: Jun 02, 2023
|
#102023
Hi,
I had circled in the end on E, is it possible for an administrator or someone to provide a more detailed reasoning behind the choice.
User avatar
 Jeff Wren
PowerScore Staff
  • PowerScore Staff
  • Posts: 389
  • Joined: Oct 19, 2022
|
#102068
Hi aadhiswaran,

The wording of Answer E is a bit tricky, but this is really the opposite of what the author believes.

Answer E is basically saying that while the pollution is harmful, fish farming will likely relieve the pressure on wild fishery stocks. In other words, fish farming will be successful in helping restore wild fishery stocks even with the pollution it causes.

The author states that "there is, however, little if any evidence that fish farming will restore ocean fishery stocks" (lines 13-14).

Then, in the second paragraph, the author discusses pollution caused by intensive fish farming and notes that such pollution can cause habitat destruction, "seriously damaging ocean and coastal resources and, ultimately, wild fishery stocks" (lines 30-31).

While the author admits that "expanding farm production does have the potential to alleviate some of the pressure on wild fishery stocks" (lines 48-49), the key words here are "potential" and "some," each denoting uncertainty.

Get the most out of your LSAT Prep Plus subscription.

Analyze and track your performance with our Testing and Analytics Package.